How Can I Find the Limit of the Natural Log Function in the Form of a Question?

In summary, we can use the Stolz-Cesaro Theorem and logarithm rules to simplify the expression for the limit and relate it to a Riemann sum. With the bounded and continuous nature of the function, we can replace the integral with the Riemann sum and ultimately find the limit to be $\frac{1}{2}$. However, further justification is needed for the replacement of the Riemann sum by the integral inside the limit.
  • #1
anemone
Gold Member
MHB
POTW Director
3,883
115
Find the

$\displaystyle\lim_{n\to\infty}\frac{2ln(2)+3ln(3)+...+nln(n)}{n^2ln(n)}$
 
Physics news on Phys.org
  • #2
anemone said:
Find the

$\displaystyle\lim_{n\to\infty}\frac{2ln(2)+3ln(3)+...+nln(n)}{n^2ln(n)}$

Hi anemone,

Use the logarithm rules, \(\ln (ab)=\ln (a)+\ln (b)\mbox{ and }\ln (a^n)=n\ln (a)\) on the numerator and try to simplify it.
 
  • #3
anemone said:
Find the

$\displaystyle\lim_{n\to\infty}\frac{2ln(2)+3ln(3)+...+nln(n)}{n^2ln(n)}$

A comfortable solution is given from the Stolz-Cesaro Theorem that extablishes, given two sequences $a_{n}$ and $b_{n}$, that...

$\displaystyle \lim_{n \rightarrow \infty} \frac{a_{n}}{b_{n}} = \lim_{n \rightarrow \infty} \frac{a_{n+1}-a_{n}}{b_{n+1}-b_{n}}$ (1)

... if the limits in (1) exist. In that case $\displaystyle a_{n}=\sum_{k=1}^{n} k\ \ln k$ and $\displaystyle b_{n}=n^{2}\ \ln n$ so that is...

$\displaystyle \lim_{n \rightarrow \infty} \frac{a_{n}}{b_{n}}= \lim_{n \rightarrow \infty} \frac{(n+1)\ \ln (n+1)}{(n+1)^{2}\ \ln (n+1) - n^{2}\ \ln n}= \lim_{n \rightarrow \infty} \frac{n+1}{2n+1}= \frac{1}{2}$ (2)

Kind regards

$\chi$ $\sigma$
 
  • #4
anemone said:
Find the

$\displaystyle\lim_{n\to\infty}\frac{2ln(2)+3ln(3)+...+nln(n)}{n^2ln(n)}$

One way of doing this is to relate the sum: \(\sum_{k=2}^n k\ln(k) \) to the integral \(\int_{x=2}^n x\ln(x)\;dx\) where we note that the integrand in the latter is an increasing function.

CB
 
  • #5
$$\begin{align*}\lim_{n \to \infty}\sum_{r=1}^{n}\frac{r \ln(r)}{n^2 \ln(n)} &= \lim_{n \to \infty} \frac{1}{\ln(n)} \left[ \sum_{r=1}^{n}\frac{r}{n^2} \left\{ \ln{\left( \frac{r}{n}\right)+\ln(n)}\right\}\right] \\ &= \lim_{n \to \infty} \frac{1}{\ln(n)} \left[ \sum_{r=1}^{n}\frac{r}{n^2}\ln{\left( \frac{r}{n}\right)}+ \sum_{r=1}^{n}\frac{r\ln(n)}{n^2}\right] \\ &=
\lim_{n \to \infty}\frac{1}{\ln(n)} \left[ \int_{0}^{1}x \cdot \ln(x) \, dx + \frac{\ln(n)}{n^2}\sum_{r=1}^{n}r\right] \\
&=\lim_{n \to \infty}\frac{1}{\ln(n)} \left[ -\frac{1}{4} + \frac{n(n+1)\ln(n)}{2n^2}\right] \\
&=\lim_{n \to \infty} \left[ -\frac{1}{4\ln(n)}+\frac{1}{2}+\frac{1}{2n}\right] \\
&=0+\frac{1}{2}+0 \\
&=\frac{1}{2}
\end{align*}$$
 
  • #6
sbhatnagar said:
$$\begin{align*}\lim_{n \to \infty}\sum_{r=1}^{n}\frac{r \ln(r)}{n^2 \ln(n)} &= \lim_{n \to \infty} \frac{1}{\ln(n)} \left[ \sum_{r=1}^{n}\frac{r}{n^2} \left\{ \ln{\left( \frac{r}{n}\right)+\ln(n)}\right\}\right] \\ &= \lim_{n \to \infty} \frac{1}{\ln(n)} \left[ \sum_{r=1}^{n}\frac{r}{n^2}\ln{\left( \frac{r}{n}\right)}+ \sum_{r=1}^{n}\frac{r\ln(n)}{n^2}\right] \\ &=
\lim_{n \to \infty}\frac{1}{\ln(n)} \left[ \int_{0}^{1}x \cdot \ln(x) \, dx + \frac{\ln(n)}{n^2}\sum_{r=1}^{n}r\right] \\
&=\lim_{n \to \infty}\frac{1}{\ln(n)} \left[ -\frac{1}{4} + \frac{n(n+1)\ln(n)}{2n^2}\right] \\
&=\lim_{n \to \infty} \left[ -\frac{1}{4\ln(n)}+\frac{1}{2}+\frac{1}{2n}\right] \\
&=0+\frac{1}{2}+0 \\
&=\frac{1}{2}
\end{align*}$$

The replacement of the Riemann sum by the integral inside the limit when going from line 2 to 3 needs more justification.

CB
 
  • #7
CaptainBlack said:
The replacement of the Riemann sum by the integral inside the limit when going from line 2 to 3 needs more justification.

CB

In $(0,1]$ the function $f(x)= x\ \ln x$ is bounded [more precisely $0 \le |f(x)| \le \frac{1}{e}$...] and continuous so that is Riemann-integrable...

Kind regards

$\chi$ $\sigma$
 
  • #8
chisigma said:
In $(0,1]$ the function $f(x)= x\ \ln x$ is bounded [more precisely $0 \le |f(x)| \le \frac{1}{e}$...] and continuous so that is Riemann-integrable...

Kind regards

$\chi$ $\sigma$

1. I know the limit/integral exists

2. I know that the method can be made to work

2. You cannot replace a finite sum by the limit, inside the limit that way, it needs further justification, not much but some: The LaTeX is now unreadable, but used something like:

\[ \lim_{n \to \infty} \left( a_n b_n \right) =\left(\lim_{n \to \infty}a_n\right)\left(\lim_{n \to \infty}b_n\right) \]

CB
 
Last edited:

Related to How Can I Find the Limit of the Natural Log Function in the Form of a Question?

What is the natural logarithm and how does it relate to limits?

The natural logarithm, denoted as ln(x), is a mathematical function that is the inverse of the exponential function. It is used to find the exponent to which a base number must be raised to obtain a given number. In the context of limits, the natural logarithm is often used to evaluate the behavior of a function as it approaches a specific value.

What is the process for finding the limit of a natural logarithm?

To find the limit of a natural logarithm, we need to substitute the value that the function is approaching into the logarithm, simplify the expression, and then take the limit as the input value approaches the specified value. In some cases, we may need to use algebraic manipulations or L'Hopital's rule to evaluate the limit.

What are the common properties of natural logarithms that are useful in finding limits?

Some of the common properties of natural logarithms that are useful in finding limits include the fact that ln(1) = 0, ln(e) = 1, and ln(ab) = ln(a) + ln(b). Additionally, the natural logarithm function is continuous and differentiable for all positive real numbers.

Are there any special cases when evaluating the limit of a natural logarithm?

Yes, there are some special cases when evaluating the limit of a natural logarithm. For example, if the input value is approaching 0, we will end up with an indeterminate form of ln(0), which cannot be evaluated. In such cases, we need to use techniques such as L'Hopital's rule or algebraic manipulations to evaluate the limit.

How can the limit of a natural logarithm be used in real-world applications?

The limit of a natural logarithm can be used in various real-world applications, such as in finance, biology, and physics. In finance, it can be used to calculate compound interest and continuously compounded interest rates. In biology, it can be used to model population growth and decay. In physics, it can be used to describe the rate of change of a quantity over time.

Similar threads

Replies
9
Views
1K
Replies
2
Views
1K
Replies
16
Views
3K
Replies
3
Views
1K
Replies
6
Views
1K
  • Calculus
Replies
3
Views
1K
Replies
24
Views
2K
Replies
6
Views
830
Back
Top